Đến nội dung

tuan101293 nội dung

Có 316 mục bởi tuan101293 (Tìm giới hạn từ 05-06-2020)



Sắp theo                Sắp xếp  

#242808 Chém gió đê ( Anh em vào cho sôi động tí)

Đã gửi bởi tuan101293 on 03-10-2010 - 18:58 trong Dãy số - Giới hạn

May man thoi :beta



#242754 Chém gió đê ( Anh em vào cho sôi động tí)

Đã gửi bởi tuan101293 on 03-10-2010 - 07:50 trong Dãy số - Giới hạn

Tiếp nào:
Tìm $ lim (2010\sqrt[n]{4}-2009)^n$

Xem nào
Đặt $S=(2010*4^\dfrac{1}{x}-2009)^x$
lấy ln 2 vế(ko bik có ra ko nữa,lấy thử :beta)
$lnS=\dfrac{ln(2010*4^\dfrac{1}{x}-2009)}{\dfrac{1}{x}}$
ặc,lim trên tử và dưới tử=0,xài thử Lopitan xem sao
$lim lnS=\dfrac{\dfrac{1}{2010*4^\dfrac{1}{x}-2009}*2010*4^\dfrac{1}{x}*ln4*\dfrac{-1}{x^2}}{\dfrac{-1}{x^2}}--->2010*ln4$
suy ra
$lim S=4^{2010}$



#242141 Đề thi chọn đội tuyển toán Nam Định 2010 - 2011

Đã gửi bởi tuan101293 on 27-09-2010 - 22:27 trong Thi HSG cấp Tỉnh, Thành phố. Olympic 30-4. Đề thi và kiểm tra đội tuyển các cấp.

Đề thi kiểu j mà ngày thi lại là 24/12/2010????????
tự chế???



#242362 Đề thi chọn đội tuyển toán Nam Định 2010 - 2011

Đã gửi bởi tuan101293 on 29-09-2010 - 20:46 trong Thi HSG cấp Tỉnh, Thành phố. Olympic 30-4. Đề thi và kiểm tra đội tuyển các cấp.

Merci.
mình khoái nhất pth mà.



#259808 Một số bài số học

Đã gửi bởi tuan101293 on 02-05-2011 - 17:02 trong Số học

Bài 3:gọi các số trong dãy là $a_i$
Xét $ \delta =\prod (2a_i-1)$
qua 1 fep biến đổi bất kỳ chú ý rằng $(2x-1)(2y-1)=2(2xy-y-x+1)-1$
nên $\delta$ là ko đổi
chú ý trong dãy có số 1/2 nên $\delta =0$ và số cuối cùng nhận được là sẽ 1/2



#242305 Đề thi chọn đội tuyển toán Nam Định 2010 - 2011

Đã gửi bởi tuan101293 on 29-09-2010 - 15:40 trong Thi HSG cấp Tỉnh, Thành phố. Olympic 30-4. Đề thi và kiểm tra đội tuyển các cấp.

Chém thử bài phương trình hàm
Đề
tìm f:R vào R thoả mãn
$f(xf(x+y))=f(yf(x))+x^2$
*******************
Dễ thấy $f(0)=0$ khi và chỉ khi x=0,$f(xf(x))=x^2,f(-xf(x))=-x^2$.
Nếu tồn tại $a>b $ mà $f(a)=f(b)$ suy ra $b^2=f(bf(a))=f(bf(b+a-b))=f((a-b)f(b))+b^2$ suy ra $(a-b)f(b)=0$ vô lý
suy ra f(x) là 1 song ánh.
chú ý $f(f(1))=1$ suy ra $1=f(f(1))=f(f(f(1))f(1))=f(1)^2$
suy ra $f(1)=1$ hoặc $f(1)=-1$
Xét f(1)=1 (TH còn lại tươg tự)
có $f(xf(2x))=2x^2=f(\sqrt{2}xf(\sqrt{2}x))$
suy ra $f(2x)=2f(x)$
thay x=1,x=2 vào pt đầu thì có
$f(f(y+1))=f(y)+1$
$f(f(y+2))=f(y)+2$ suy ra $f(f(y+1))=f(f(y))+1$
suy ra $f(f(y))=f(y)$ suy ra $f(x)=x$ với mọi x.
Trả lời f(x)=x,f(x)=-x



#220921 bất đẳng thức c/m bằng quy nạp nè

Đã gửi bởi tuan101293 on 20-11-2009 - 08:49 trong Bất đẳng thức và cực trị

có bài này ai rảnh trả lời nha ( có trong toán tuoi tho )
so sánh $ 1999^{1998}$ và $ 1998^{1999} $
bít kq nhưng quên mât cách làm

ta có $1998^{1999}>1999^{1998}$
bạn có thể CM như sau
xét hàm $f(x)=\sqrt[x]{x}$
$f'(x)=\sqrt[x]{x}*(\dfrac{1-ln(x)}{x^2})<0$ với x>3
suy ra $\sqrt[1998]{1998}>\sqrt[1999]{1999}$ .....



#244799 1 bài toán khó

Đã gửi bởi tuan101293 on 22-10-2010 - 19:35 trong Các bài toán Đại số khác

Em đọc kỹ xem,đúng đó,ko cần chia nhiều TH đâu:D



#248884 Chứng minh

Đã gửi bởi tuan101293 on 10-12-2010 - 12:08 trong Bất đẳng thức và cực trị

Cho tam giác ABC điểm M nằm trong tam giác!
Gọi $ MA=R_{a}, MB=R_{b},MC=R_{c}$ , $d_{a},d_{b}, d_{c}$ lần lượt là khoảng cách từ M đến BC, AC, AB chứng minh
$ R_{a}+ R_{b}+R_{c} \geq 2(d_{a}+d_{b}+ d_{c})$
Sao không ai nhớ he??

Đây là bdt Erdos
Cho chú mười mấy cách CM này

File gửi kèm




#454393 $u_{1}=1;u_{2}=2;u_{n+1}=\dfrac{...

Đã gửi bởi tuan101293 on 01-10-2013 - 05:31 trong Dãy số - Giới hạn

Theo em thì có lẽ bài này sai đề, với lại nếu đúng đi nữa thì số hạng thứ 5 cũng không thuộc N nên có lẽ không cần quan tâm cho mệt @@




#307640 China TST 2012

Đã gửi bởi tuan101293 on 01-04-2012 - 21:23 trong Thi HSG Quốc gia và Quốc tế

Xét số $k$ bất kì có ước nguyên tố là $p_n$.
Nếu số $m$ là một số tốt,không chia hết cho $k$ và $m$ có ít nhất một ước nguyên tố $p_l$ với $l>n$ khi đó số $m.\frac{p_n}{p_l}<m$ và có số ước không ít hơn số ước của $m$. Dẫn đến mâu thuẫn.

mình không rõ đoạn này lắm, nếu $p_{n}$ là ước của $m$ thì sao ???



#252124 nguyên hàm đây zô cho zui

Đã gửi bởi tuan101293 on 25-01-2011 - 21:59 trong Tích phân - Nguyên hàm

Không ai ủng hộ topic của tui sao buôn quá trời. Hay là chưa đủ khó để làm vậy thì tiếp nè .....

1.Nguyên hàm $\int {\left( {\dfrac{1}{{\ln ^2 }} + \dfrac{1}{{\ln x}}} \right)} dx$

2.$\int {\dfrac{1}{{\ln ^2 }}dx} $ và $\int {\dfrac{1}{{\ln x}}dx} $

Mình chỉ biết cái nguyên hàm này thôi

$ \int (\dfrac{1}{ln(x)}-\dfrac{1}{ln(x)^2})dx =\dfrac{x}{ln(x)}$
mấy cái kia có lẽ là ko tính được đâu



#248981 Chứng minh

Đã gửi bởi tuan101293 on 12-12-2010 - 09:19 trong Bất đẳng thức và cực trị

Bị lỗi anh ơi!
Anh gư lại xem nào???

lại này

File gửi kèm




#244740 1 bài toán khó

Đã gửi bởi tuan101293 on 22-10-2010 - 14:19 trong Các bài toán Đại số khác

Cho f là hàm số lẻ. Hàm số ĐB trên R.
Nếu a+b+c=0. Chứng minh rằng

f(a).f(b)+f(b).f(c )+f(c ).f(a) :Rightarrow 0
Chứng minh giùm em với!

Em đang cần rất gấp

+, f là hàm lẻ suy ra $f(x)=-f(-x)$ nên f(0)=0
+, Giả sử $a\ge b\ge c$
suy ra ta phải CM
$f(a)f(b)\le (f(b)+f(a))f(a+b)$
Ta có $f(a)+f(b)\ge 0$
và $f(a+b)\ge f(b)$ do $a\ge 0$
suy ra $(f(a)+f(b))f(a+b)\ge (f(a)+f(b))f(b)\ge f(a)f(b)$
ĐPCM



#244201 bai kho

Đã gửi bởi tuan101293 on 18-10-2010 - 20:58 trong Bất đẳng thức và cực trị

Ý tưởng của dạng này có lẽ là loại bỏ trường hợp hiển nhiên,trường hợp còn lại xài bdt Schur.
Ví dụ bài trên nhé
TH1:$\sum a^2>2\sum ab$,hiển nhiên
TH2:$\sum a^2\le 2\sum ab$
Theo Schur ta có $2\sum ab-\sum a^2\le \dfrac{9abc}{a+b+c}\le 2abc+1$ (côsi)
ĐPCM



#226354 bất đẳng thức tự sáng tạo

Đã gửi bởi tuan101293 on 17-01-2010 - 08:28 trong Bất đẳng thức - Cực trị

Bài của bạn tùng làm thế này:
$VT-VP=\dfrac{(\sum a)(\sum a^2-\sum bc)}{3abc}+\dfrac{(\sum bc-\sum a^2)(\sum bc+\sum a^2)}{(\sum a^2)^2}=(\sum a^2-\sum bc)*S$
trong đó
$S=\dfrac{\sum a}{3abc}-\dfrac{\sum a^2+\sum bc}{(\sum a^2)^2}\ge \dfrac{3}{(\sum a^2)}-\dfrac{2\sum a^2}{(\sum a^2)^2}>0$
p/s: từ cách này thì ta có thể thay số 3 bằng số 9/2 (làm mạnh bdt)



#244255 bai kho

Đã gửi bởi tuan101293 on 19-10-2010 - 12:54 trong Bất đẳng thức và cực trị

anh co the dung may ki hieu cua trung hoc co so de giai duoc ko ?
may cai kia bon em chua hoc !

OK,
Ta phải CM
$a^2+b^2+c^2+2abc+1\ge 2(ab+bc+ca)$
Th1:$a^2+b^2+c^2>2(ab+bc+ca)$
bdt hiển nhiên đúng
Th2:$a^2+b^2+c^2\le 2(ab+bc+ca)$
theo bdt schur ta có
$2(ab+bc+ca)-a^2-b^2-c^2\le \dfrac{9abc}{a+b+c}\le \dfrac{9abc}{3\sqrt[3]{abc}}=3\sqrt[3]{a^2b^2c^2}\le abc+abc+1=2abc+1$
ĐPCM



#272897 Số nguyên tố

Đã gửi bởi tuan101293 on 17-08-2011 - 23:06 trong Số học

sao $p|q-1$ đựoc nhỉ?

mình đánh nhầm p với q
p là cấp của m modulo q nên p|q-1
********
chú ý ở đầu ta xài bổ đề
$(\dfrac{a^n-b^n}{a-b},a-b)=(a-b,n)$



#241647 tìm CTTQ của dãy số ?

Đã gửi bởi tuan101293 on 22-09-2010 - 19:30 trong Dãy số - Giới hạn

nhưng tớ ko hiểu tại sao đến U 5 là không xác định rui sao có thể đặt tan được ???

Bạn nói đúng 1 phần :(
Làm theo cách mình thì $u_n=tan(\dfrac{\pi}{6}+(n-1)*\dfrac{\pi}{12})$ khi đó thì $u_5=tan(\dfrac{\pi}{2})=\infty $ là ko xác định
Nhưng nếu bạn tính = tay thì đến $u_2=1,u_3=\sqrt{3},u_4=2+\sqrt{3}$ và đến $u_5$ thì cũng ko xác định .
(Cái này là do đề bẫy thôi bạn à,phương pháp vẫn thế)



#272846 Số nguyên tố

Đã gửi bởi tuan101293 on 17-08-2011 - 19:51 trong Số học

Đề bài: Chứng minh có vô số số nguyên tố có dạng 2pn +1 với p là số nguyên tố lẻ bất kì.

Xét với số p tùy ý thuộc P, lấy m tùy ý mà (m-1) ko chia hết cho p
xét q thuộc P mà $q|\dfrac{m^p-1}{m-1}$
ta có $(\dfrac{m^p-1}{m-1},m-1)=1$ nên p là cấp của m modulo q hay
$p|q-1$ hay $q=k_m*p+1$
đpcm



#238986 Nice

Đã gửi bởi tuan101293 on 01-09-2010 - 19:59 trong Bất đẳng thức và cực trị

$a,b,c>0$
$ab+bc+ca=3$
Chung minh: $ \sum \dfrac{a}{2 a^{2}+bc } \geq abc$

đẹp và dễ thật :)
ta có bdt tương đương (chia abc cho 2 vế)
$\sum \dfrac{1}{2a^2bc+b^2c^2}\ge 1$ đặt ab=z,bc=x,ca=y suy ra x+y+z=3
và ta phải CM
$\sum \dfrac{1}{x^2+2yz}\ge \dfrac{9}{x^2+y^2+z^2+2xy+2yz+2zx}=1$
ĐPCM



#241574 tìm CTTQ của dãy số ?

Đã gửi bởi tuan101293 on 21-09-2010 - 17:40 trong Dãy số - Giới hạn

$ {U_1} = \dfrac{1}{{\sqrt 3 }} $
${U_{n + 1}} = \dfrac{{{U_n} + 2 - \sqrt 3 }}{{1 + \sqrt 3 {U_n}}} - 2{U_n} $

Đề như thế này à?

Dịch sai đề rồi
bài này nhìn kỹ thì nó phải thía này :
${U_{n + 1}} = \dfrac{{{U_n} + 2 - \sqrt 3 }}{{1 + \sqrt 3 {U_n}} - 2{U_n}} $
kiều này dạng $\dfrac{a+b}{1-ab}$.quy về cái tan là ok thôi.



#224173 Một vài bài bdt lượng giác

Đã gửi bởi tuan101293 on 30-12-2009 - 12:04 trong Bất đẳng thức - Cực trị

bài 1: $sin \dfrac{A}{2}+sin \dfrac{B}{2}+sin \dfrac{C}{2} \leq 1+ \dfrac{1}{2}cos(B-C)$
bài 2: $tan^{2} \dfrac{A}{2} + tan^{2} \dfrac{B}{2} + tan^{2} \dfrac{C}{2} +2(cosA+cosB+cosC) \geq 4$

bài 1 cho A=C tiến đến 0,B tiến đến 180 thì sai.(nếu đúng thì chắc ý tưởng chỉ là tam thức bậc 2)



#241600 tìm CTTQ của dãy số ?

Đã gửi bởi tuan101293 on 21-09-2010 - 23:16 trong Dãy số - Giới hạn

Can moi so hang dau thoi mah.??



#258981 Giair một số BĐT

Đã gửi bởi tuan101293 on 25-04-2011 - 06:36 trong Bất đẳng thức - Cực trị

1,ta có $x^3+y^3=(x+y)(x^2-xy+y^2)\ge (x+y)\dfrac{(x^2+xy+y^2)}{3}$
suy ra
$3VT\ge \sum\dfrac{(z+y)(z^2+zy+y^2)}{x^2+xy+y^2}\ge 3\sqrt[3]{\prod (x+y)}\ge 6$
suy ra $VT\ge 2$
ĐPCM